If $Z$ is $sigma(X,Y)$-measurable, is there a measurable $f$ with $Z=f(X,Y)$?











up vote
0
down vote

favorite












Let





  • $(Omega_i,mathcal A_i)$ be a measurable space

  • $X:Omega_1toOmega_2$

  • $Z:Omega_1tomathbb R$


It's easy to show that $Z$ is $sigma(X)$-measurable if and only if there is a $mathcal A_2$-measurable $f:Omega_2tomathbb R$ with $$Z=f(X).tag1$$ Now, suppose $Y:Omega_1toOmega_3$. Are we able to show that if $Z$ is $sigma(X,Y)$-measurable, then there is a $mathcal A_2otimesmathcal A_3$-measurable $g:Omega_2timesOmega_3tomathbb R$ with $$Z=g(X,Y)tag2?$$



(By the way: Is it possible to generalize $(1)$ to Banach space valued strongly measurable $Z,f$?)










share|cite|improve this question


























    up vote
    0
    down vote

    favorite












    Let





    • $(Omega_i,mathcal A_i)$ be a measurable space

    • $X:Omega_1toOmega_2$

    • $Z:Omega_1tomathbb R$


    It's easy to show that $Z$ is $sigma(X)$-measurable if and only if there is a $mathcal A_2$-measurable $f:Omega_2tomathbb R$ with $$Z=f(X).tag1$$ Now, suppose $Y:Omega_1toOmega_3$. Are we able to show that if $Z$ is $sigma(X,Y)$-measurable, then there is a $mathcal A_2otimesmathcal A_3$-measurable $g:Omega_2timesOmega_3tomathbb R$ with $$Z=g(X,Y)tag2?$$



    (By the way: Is it possible to generalize $(1)$ to Banach space valued strongly measurable $Z,f$?)










    share|cite|improve this question
























      up vote
      0
      down vote

      favorite









      up vote
      0
      down vote

      favorite











      Let





      • $(Omega_i,mathcal A_i)$ be a measurable space

      • $X:Omega_1toOmega_2$

      • $Z:Omega_1tomathbb R$


      It's easy to show that $Z$ is $sigma(X)$-measurable if and only if there is a $mathcal A_2$-measurable $f:Omega_2tomathbb R$ with $$Z=f(X).tag1$$ Now, suppose $Y:Omega_1toOmega_3$. Are we able to show that if $Z$ is $sigma(X,Y)$-measurable, then there is a $mathcal A_2otimesmathcal A_3$-measurable $g:Omega_2timesOmega_3tomathbb R$ with $$Z=g(X,Y)tag2?$$



      (By the way: Is it possible to generalize $(1)$ to Banach space valued strongly measurable $Z,f$?)










      share|cite|improve this question













      Let





      • $(Omega_i,mathcal A_i)$ be a measurable space

      • $X:Omega_1toOmega_2$

      • $Z:Omega_1tomathbb R$


      It's easy to show that $Z$ is $sigma(X)$-measurable if and only if there is a $mathcal A_2$-measurable $f:Omega_2tomathbb R$ with $$Z=f(X).tag1$$ Now, suppose $Y:Omega_1toOmega_3$. Are we able to show that if $Z$ is $sigma(X,Y)$-measurable, then there is a $mathcal A_2otimesmathcal A_3$-measurable $g:Omega_2timesOmega_3tomathbb R$ with $$Z=g(X,Y)tag2?$$



      (By the way: Is it possible to generalize $(1)$ to Banach space valued strongly measurable $Z,f$?)







      probability-theory measure-theory measurable-functions






      share|cite|improve this question













      share|cite|improve this question











      share|cite|improve this question




      share|cite|improve this question










      asked Nov 16 at 22:11









      0xbadf00d

      1,86041428




      1,86041428






















          1 Answer
          1






          active

          oldest

          votes

















          up vote
          0
          down vote













          If $Z=I_{X^{-1}(A)cap Y^{-1}(B)}$ then $Z=f(X,Y)$ where $f=I_{Atimes B}$. Now ${Cin mathcal B(mathbb R^{2}):I_{{(X,Y)^{-1}}(C)}=f(X,Y) text {for some measurable} f:mathbb R^{2} to mathbb R}$ is a sigma algebra which contains measurable rectangles so it contains all Borel sets in $mathbb R^{2}$. It follow now that the result is true for any simple function $Z$ measurable w.r.t. $sigma (X,Y)$. Hence the same holde for non-negative meaurble functions. If $Z_n=f_n(X,Y)$ for all $n$ and $Z_n to Z$ then $Z=limsup f_n(X,Y)$. Now write $Z$ as $Z^{+}-Z^{-}$ to complete the proof. The same argument works for strongly measurable Banach valued $Z$.






          share|cite|improve this answer























          • My problem with the Banach space case is that there is no $operatorname{lim sup}$.
            – 0xbadf00d
            Nov 17 at 10:49










          • Something is wrong with your definition of your $sigma$-algebra. Why $Cinmathcal B(mathbb R^2)$?
            – 0xbadf00d
            Nov 17 at 10:54










          • You are right. I typed the definition wrongly. Please see the revised answer. @Oxbadf00d
            – Kavi Rama Murthy
            Nov 17 at 11:36













          Your Answer





          StackExchange.ifUsing("editor", function () {
          return StackExchange.using("mathjaxEditing", function () {
          StackExchange.MarkdownEditor.creationCallbacks.add(function (editor, postfix) {
          StackExchange.mathjaxEditing.prepareWmdForMathJax(editor, postfix, [["$", "$"], ["\\(","\\)"]]);
          });
          });
          }, "mathjax-editing");

          StackExchange.ready(function() {
          var channelOptions = {
          tags: "".split(" "),
          id: "69"
          };
          initTagRenderer("".split(" "), "".split(" "), channelOptions);

          StackExchange.using("externalEditor", function() {
          // Have to fire editor after snippets, if snippets enabled
          if (StackExchange.settings.snippets.snippetsEnabled) {
          StackExchange.using("snippets", function() {
          createEditor();
          });
          }
          else {
          createEditor();
          }
          });

          function createEditor() {
          StackExchange.prepareEditor({
          heartbeatType: 'answer',
          convertImagesToLinks: true,
          noModals: true,
          showLowRepImageUploadWarning: true,
          reputationToPostImages: 10,
          bindNavPrevention: true,
          postfix: "",
          imageUploader: {
          brandingHtml: "Powered by u003ca class="icon-imgur-white" href="https://imgur.com/"u003eu003c/au003e",
          contentPolicyHtml: "User contributions licensed under u003ca href="https://creativecommons.org/licenses/by-sa/3.0/"u003ecc by-sa 3.0 with attribution requiredu003c/au003e u003ca href="https://stackoverflow.com/legal/content-policy"u003e(content policy)u003c/au003e",
          allowUrls: true
          },
          noCode: true, onDemand: true,
          discardSelector: ".discard-answer"
          ,immediatelyShowMarkdownHelp:true
          });


          }
          });














           

          draft saved


          draft discarded


















          StackExchange.ready(
          function () {
          StackExchange.openid.initPostLogin('.new-post-login', 'https%3a%2f%2fmath.stackexchange.com%2fquestions%2f3001694%2fif-z-is-sigmax-y-measurable-is-there-a-measurable-f-with-z-fx-y%23new-answer', 'question_page');
          }
          );

          Post as a guest















          Required, but never shown

























          1 Answer
          1






          active

          oldest

          votes








          1 Answer
          1






          active

          oldest

          votes









          active

          oldest

          votes






          active

          oldest

          votes








          up vote
          0
          down vote













          If $Z=I_{X^{-1}(A)cap Y^{-1}(B)}$ then $Z=f(X,Y)$ where $f=I_{Atimes B}$. Now ${Cin mathcal B(mathbb R^{2}):I_{{(X,Y)^{-1}}(C)}=f(X,Y) text {for some measurable} f:mathbb R^{2} to mathbb R}$ is a sigma algebra which contains measurable rectangles so it contains all Borel sets in $mathbb R^{2}$. It follow now that the result is true for any simple function $Z$ measurable w.r.t. $sigma (X,Y)$. Hence the same holde for non-negative meaurble functions. If $Z_n=f_n(X,Y)$ for all $n$ and $Z_n to Z$ then $Z=limsup f_n(X,Y)$. Now write $Z$ as $Z^{+}-Z^{-}$ to complete the proof. The same argument works for strongly measurable Banach valued $Z$.






          share|cite|improve this answer























          • My problem with the Banach space case is that there is no $operatorname{lim sup}$.
            – 0xbadf00d
            Nov 17 at 10:49










          • Something is wrong with your definition of your $sigma$-algebra. Why $Cinmathcal B(mathbb R^2)$?
            – 0xbadf00d
            Nov 17 at 10:54










          • You are right. I typed the definition wrongly. Please see the revised answer. @Oxbadf00d
            – Kavi Rama Murthy
            Nov 17 at 11:36

















          up vote
          0
          down vote













          If $Z=I_{X^{-1}(A)cap Y^{-1}(B)}$ then $Z=f(X,Y)$ where $f=I_{Atimes B}$. Now ${Cin mathcal B(mathbb R^{2}):I_{{(X,Y)^{-1}}(C)}=f(X,Y) text {for some measurable} f:mathbb R^{2} to mathbb R}$ is a sigma algebra which contains measurable rectangles so it contains all Borel sets in $mathbb R^{2}$. It follow now that the result is true for any simple function $Z$ measurable w.r.t. $sigma (X,Y)$. Hence the same holde for non-negative meaurble functions. If $Z_n=f_n(X,Y)$ for all $n$ and $Z_n to Z$ then $Z=limsup f_n(X,Y)$. Now write $Z$ as $Z^{+}-Z^{-}$ to complete the proof. The same argument works for strongly measurable Banach valued $Z$.






          share|cite|improve this answer























          • My problem with the Banach space case is that there is no $operatorname{lim sup}$.
            – 0xbadf00d
            Nov 17 at 10:49










          • Something is wrong with your definition of your $sigma$-algebra. Why $Cinmathcal B(mathbb R^2)$?
            – 0xbadf00d
            Nov 17 at 10:54










          • You are right. I typed the definition wrongly. Please see the revised answer. @Oxbadf00d
            – Kavi Rama Murthy
            Nov 17 at 11:36















          up vote
          0
          down vote










          up vote
          0
          down vote









          If $Z=I_{X^{-1}(A)cap Y^{-1}(B)}$ then $Z=f(X,Y)$ where $f=I_{Atimes B}$. Now ${Cin mathcal B(mathbb R^{2}):I_{{(X,Y)^{-1}}(C)}=f(X,Y) text {for some measurable} f:mathbb R^{2} to mathbb R}$ is a sigma algebra which contains measurable rectangles so it contains all Borel sets in $mathbb R^{2}$. It follow now that the result is true for any simple function $Z$ measurable w.r.t. $sigma (X,Y)$. Hence the same holde for non-negative meaurble functions. If $Z_n=f_n(X,Y)$ for all $n$ and $Z_n to Z$ then $Z=limsup f_n(X,Y)$. Now write $Z$ as $Z^{+}-Z^{-}$ to complete the proof. The same argument works for strongly measurable Banach valued $Z$.






          share|cite|improve this answer














          If $Z=I_{X^{-1}(A)cap Y^{-1}(B)}$ then $Z=f(X,Y)$ where $f=I_{Atimes B}$. Now ${Cin mathcal B(mathbb R^{2}):I_{{(X,Y)^{-1}}(C)}=f(X,Y) text {for some measurable} f:mathbb R^{2} to mathbb R}$ is a sigma algebra which contains measurable rectangles so it contains all Borel sets in $mathbb R^{2}$. It follow now that the result is true for any simple function $Z$ measurable w.r.t. $sigma (X,Y)$. Hence the same holde for non-negative meaurble functions. If $Z_n=f_n(X,Y)$ for all $n$ and $Z_n to Z$ then $Z=limsup f_n(X,Y)$. Now write $Z$ as $Z^{+}-Z^{-}$ to complete the proof. The same argument works for strongly measurable Banach valued $Z$.







          share|cite|improve this answer














          share|cite|improve this answer



          share|cite|improve this answer








          edited Nov 17 at 11:36

























          answered Nov 16 at 23:21









          Kavi Rama Murthy

          42.8k31751




          42.8k31751












          • My problem with the Banach space case is that there is no $operatorname{lim sup}$.
            – 0xbadf00d
            Nov 17 at 10:49










          • Something is wrong with your definition of your $sigma$-algebra. Why $Cinmathcal B(mathbb R^2)$?
            – 0xbadf00d
            Nov 17 at 10:54










          • You are right. I typed the definition wrongly. Please see the revised answer. @Oxbadf00d
            – Kavi Rama Murthy
            Nov 17 at 11:36




















          • My problem with the Banach space case is that there is no $operatorname{lim sup}$.
            – 0xbadf00d
            Nov 17 at 10:49










          • Something is wrong with your definition of your $sigma$-algebra. Why $Cinmathcal B(mathbb R^2)$?
            – 0xbadf00d
            Nov 17 at 10:54










          • You are right. I typed the definition wrongly. Please see the revised answer. @Oxbadf00d
            – Kavi Rama Murthy
            Nov 17 at 11:36


















          My problem with the Banach space case is that there is no $operatorname{lim sup}$.
          – 0xbadf00d
          Nov 17 at 10:49




          My problem with the Banach space case is that there is no $operatorname{lim sup}$.
          – 0xbadf00d
          Nov 17 at 10:49












          Something is wrong with your definition of your $sigma$-algebra. Why $Cinmathcal B(mathbb R^2)$?
          – 0xbadf00d
          Nov 17 at 10:54




          Something is wrong with your definition of your $sigma$-algebra. Why $Cinmathcal B(mathbb R^2)$?
          – 0xbadf00d
          Nov 17 at 10:54












          You are right. I typed the definition wrongly. Please see the revised answer. @Oxbadf00d
          – Kavi Rama Murthy
          Nov 17 at 11:36






          You are right. I typed the definition wrongly. Please see the revised answer. @Oxbadf00d
          – Kavi Rama Murthy
          Nov 17 at 11:36




















           

          draft saved


          draft discarded



















































           


          draft saved


          draft discarded














          StackExchange.ready(
          function () {
          StackExchange.openid.initPostLogin('.new-post-login', 'https%3a%2f%2fmath.stackexchange.com%2fquestions%2f3001694%2fif-z-is-sigmax-y-measurable-is-there-a-measurable-f-with-z-fx-y%23new-answer', 'question_page');
          }
          );

          Post as a guest















          Required, but never shown





















































          Required, but never shown














          Required, but never shown












          Required, but never shown







          Required, but never shown

































          Required, but never shown














          Required, but never shown












          Required, but never shown







          Required, but never shown







          Popular posts from this blog

          How do I know what Microsoft account the skydrive app is syncing to?

          Grease: Live!

          When does type information flow backwards in C++?